IMO Shortlist 1989 problem 20


Kvaliteta:
  Avg: 0,0
Težina:
  Avg: 0,0
Dodao/la: arhiva
2. travnja 2012.
LaTeX PDF
Let n and k be positive integers and let S be a set of n points in the plane such that

i.) no three points of S are collinear, and

ii.) for every point P of S there are at least k points of S equidistant from P.

Prove that:
k < \frac {1}{2} + \sqrt {2 \cdot n}
Izvor: Međunarodna matematička olimpijada, shortlist 1989